From: W. Trevor King Date: Mon, 30 Apr 2012 19:33:30 +0000 (-0400) Subject: Fix solution rounding in Serway and Jewett v8's problem 27.1. X-Git-Url: http://git.tremily.us/?p=course.git;a=commitdiff_plain;h=7cbe267e0658afcbec1c0b83f473d3fd8e0a2840 Fix solution rounding in Serway and Jewett v8's problem 27.1. --- diff --git a/latex/problems/Serway_and_Jewett_8/problem27.01.tex b/latex/problems/Serway_and_Jewett_8/problem27.01.tex index 27bbf43..2e738ba 100644 --- a/latex/problems/Serway_and_Jewett_8/problem27.01.tex +++ b/latex/problems/Serway_and_Jewett_8/problem27.01.tex @@ -13,6 +13,6 @@ where $q=1.60\E{-19}\U{C}$ is the charge of a single proton. The number of strikes in the allotted time is thus \begin{equation} \Delta N = \deriv{t}{N} \cdot \Delta t = \frac{I\Delta t}{q} - = \ans{17.9\E{15}\U{protons} = 17.9\U{petaprotons}} \;. + = \ans{18.0\E{15}\U{protons} = 18.0\U{petaprotons}} \;. \end{equation} \end{solution}